Why Is the Y-Axis the Major Axis When b > a in an Ellipse Equation?

Click For Summary
SUMMARY

In the context of the ellipse equation \(\frac{x^2}{a^2}+\frac{y^2}{b^2}=1\), when \(b > a\), the y-axis serves as the major axis. This is because the major axis is defined as the longest diameter of the ellipse, which corresponds to the maximum value between \(a\) and \(b\). Thus, the distance \(r\) from the center to any point on the ellipse is constrained by the values of \(a\) and \(b\), confirming that the major axis is always represented by the larger of the two values.

PREREQUISITES
  • Understanding of ellipse equations and their standard forms
  • Familiarity with the concepts of major and minor axes
  • Basic knowledge of coordinate geometry
  • Ability to interpret mathematical inequalities
NEXT STEPS
  • Study the properties of ellipses in conic sections
  • Learn about the geometric interpretations of major and minor axes
  • Explore the derivation of the ellipse equation from its definition
  • Investigate the applications of ellipses in physics and engineering
USEFUL FOR

Students of mathematics, educators teaching geometry, and anyone interested in the properties of conic sections will benefit from this discussion.

StopWatch
Messages
35
Reaction score
0
This is a really dumb question, but could someone quickly explain why it is that if b > a in the equation of an ellipse then y is the major axis. Just intuitively I want to think that y^2/5 as opposed to y^2/3 is going to be smaller for a given value of y since each value is being limited by the dividend. The opposite is true. Can someone explain this simply?
 
Physics news on Phys.org
\frac{x^2}{a^2}+\frac{y^2}{b^2}=1

The distance r of every point of the ellipses from the centre of the frame of reference is always between min(a,b)\leq r\leq max(a,b).
By definition, the major axis is defined as max(a,b) while the minor axis is min(a,b).
 
Question: A clock's minute hand has length 4 and its hour hand has length 3. What is the distance between the tips at the moment when it is increasing most rapidly?(Putnam Exam Question) Answer: Making assumption that both the hands moves at constant angular velocities, the answer is ## \sqrt{7} .## But don't you think this assumption is somewhat doubtful and wrong?

Similar threads

  • · Replies 8 ·
Replies
8
Views
2K
  • · Replies 5 ·
Replies
5
Views
3K
Replies
3
Views
1K
Replies
4
Views
3K
Replies
2
Views
2K
  • · Replies 1 ·
Replies
1
Views
1K
  • · Replies 6 ·
Replies
6
Views
1K
  • · Replies 14 ·
Replies
14
Views
3K
  • · Replies 2 ·
Replies
2
Views
8K
Replies
5
Views
2K